0
$\begingroup$

I have the following problem:

$\max_{z_{i,j,v}} \sum_{i}\sum_{j}\sum_{v} z_{i,j,v}$

$s.t. \sum_{i}\sum_{j} z_{i,j,v} = \max_{i,j} z_{i,j,v}$

$z_{i,j,v}\in[0,1]$

I can prove the constraint is quasilinear constraint, and the problem is convex peoblem(other constraint are omitted due to its obviously convexity). Yet, I can't handle the quasilinear constraint. I know it should be transformed in convex or affine terms, I don't know how to achieve it. So that's my question.

The meaning of the constraint is that only one $z_{i',j',v}$ can have non-zero value, which is also the maximum of z, the others $z_{i,j,v}, \forall i\neq i', j\neq j'$ should be zero. I am also not sure if there are better formulation of this meaning. I got an answer from cvxr.com, where a gentle man told me to introduce a binary vector to represent this constraint. I don't want to formulate this problem into a NP-complete one.

Here is my thinking. I can relax the constraint into the following inequality:

$\sum_{i}\sum_{j} z_{i,j,v} \leq \max_{i,j} z_{i,j,v}$

Note that it has to be $\textbf{less than }$or equal to, because the sum of z great than the maximum value naturaly. This inequality isn't a standard form because the -max function is concave, thus I have to deal with the -max function, the constraint can be decoupled into two constraints:

$\sum_{i}\sum_{j}z_{i,j,v} \leq t_v (1)$

$t_v \leq \max_{i,j} z_{i,j,v} (2)$

The difficulty is because the non-convexity of constraint (2), in fact, it is a $\textbf{reverse convex constraint}$. So should I keep going to deal with the reverse convex constraint? I am not sure, because the original constraint is in fact convex, after the tranformation, constraint (2) become non-convex.

So is there anyone can give some advice about this problem? Any suggestions will be grateful.

$\endgroup$
3
  • $\begingroup$ Maybe there is some structure in your overall problem that you can use. In general, such a transformation is not possible. $\endgroup$
    – LinAlg
    Mar 12, 2018 at 14:08
  • $\begingroup$ You're not going to get an answer here that is any different than you received there. The gentlemen on cvxr.com is correct. You cannot do better. $\endgroup$ Mar 13, 2018 at 2:52
  • $\begingroup$ Thanks for your answers. I am sorry I don't know how to response to one of you. I will rethink about to adopt the gentlemen's advice.Besides, do anyone of you can give some advice about approximate this constraint? $\endgroup$
    – WEI Xing
    Mar 13, 2018 at 7:51

0

You must log in to answer this question.

Browse other questions tagged .